LSAT and Law School Admissions Forum

Get expert LSAT preparation and law school admissions advice from PowerScore Test Preparation.

 Administrator
PowerScore Staff
  • PowerScore Staff
  • Posts: 8916
  • Joined: Feb 02, 2011
|
#40379
Complete Question Explanation
(The complete setup for this game can be found here: lsat/viewtopic.php?t=13080)

The correct answer choice is (B)

This is the infamous Rule Substitution question, a staple in the Logic Games section as of late. Considering their growing popularity, tackling such questions is no longer merely optional: it is imperative to have a workable strategy in place.
The question stem asks you to replace the first rule of the game, which required R to be placed either first or second:
PT72_Game_#2_#12_diagram 1.png
Our job is to substitute that rule with a logically equivalent condition, which would have the exact same effect on the order in which the houses are shown. While the wording of the correct answer can be difficult-to-impossible to predict, the four incorrect answer choices will either present rules that were not part of the original rule set (also known as Additional Effects conditions), or else they will only partially constrain the variables in the rule being substituted (i.e. Partial Match conditions).

Since Additional Effects are easier to spot, look for answer choices that contain such conditions first. You should immediately notice, for instance, that answer choices (D) adds a condition extraneous to the original rule set. It stipulates that Q and R must be shown consecutively, which is not required by the original rule set (see Templates 2.1 and 2.2).

We are left with answer choices (A), (B), (C), and (E).

According to answer choice (A), R cannot be shown fourth. This condition is true given our original diagram, but it does not prevent R from being fifth (which the original rule did). Therefore, answer choice (A) is a Partial Match condition, as it only partially constrains R in the rule being substituted.

Answer choice (C) is also incorrect. While the original rules did require V to be third or fourth (see Templates), this condition alone does not force R into one of the first two positions. It does if V were fourth, because then R would be forced into the second position; however, if V were third, then R could end up fourth or fifth. Therefore, answer choice (C) also contains a Partial Match, and must be eliminated.

Answer choice (E) states that if T is not fifth, then it must be shown immediately before R:
PT72_Game_#2_#12_diagram 2.png
Since T can only be shown either first or fifth, the rule can be represented as follows:
PT72_Game_#2_#12_diagram 3.png
While this is clearly an inference we can draw from the original set of solutions, it does not necessarily force R into one of the first two positions. This is because the rule does not specify what must happen to R if T were fifth. For instance, answer choice (E) would allow for the following solution to exist, which would be forbidden by the original rule being substituted:
PT72_Game_#2_#12_diagram 4.png
Thus, answer choice (E) contains a Partial Match, and must be eliminated.

Answer choice (B) is the correct answer choice. The condition requiring R to be shown earlier than V is consistent with the original diagram, as shown in the templates. Answer choice (B) does not therefore contain an Additional Effects rule. The better question is: does answer choice (B) contain a Partial Match? It does not. Here’s why:

Recall that either V or Q must be third. Well, if R is shown earlier than V, and V is third, then R must be either first or second:

..... 1. ..... If R :longline: V, and V is third:
PT72_Game_#2_#12_diagram 5.png
But what if Q is third? In that case, S and T must alternate between positions 1 and 5, in compliance with the second and fourth rules. The two remaining variables—R and V—must go to positions 2 and 4, respectively, in compliance with the condition stipulated in answer choice (B):

..... 2. ..... If R ..... V, and Q is third:
PT72_Game_#2_#12_diagram 6.png
In either case—whether Q or V is third—answer choice (B) requires R to be either first or second. Thus, the effect of the new condition is identical to that of the original rule, and answer choice (B) is correct.
You do not have the required permissions to view the files attached to this post.
 LustingFor!L
  • Posts: 80
  • Joined: Aug 27, 2016
|
#31708
Was there an easier way to approach this question aside from going through and testing each answer choice?

Thank you!
 Kristina Moen
PowerScore Staff
  • PowerScore Staff
  • Posts: 230
  • Joined: Nov 17, 2016
|
#31947
Hi LustingFor!L,

Rule Substitution questions often take the longest to complete. If you running low on time, then these are the questions to skip! But if you decide to do these questions, know they will just take a longer time. Logic Games sections should be of comparable difficulty from test to test - so if there are many Rule Substitution questions, then you'll have some easier questions too (like #7!).

That said, when approaching a Rule Substitution question like this one, you can first identify what you want. Here, I want a rule that will result in R being shown 1st or 2nd. So I want R to be constricted. How can I do this? Fill the other slots so the only two slots left are 1 and 2, make R go before some other variable that goes 3rd, and perhaps some other options. I will take a second to look at the remaining rules and see if anything jumps out. Here, there were so many options it seemed best just to go to the answer choices and try them. But by the last question, you should be fairly comfortable with coming up with scenarios for this game!

So I think the fastest way to approach this question was to test the answer choices. And luckily, the correct answer choice was (B) so you could move on!

Get the most out of your LSAT Prep Plus subscription.

Analyze and track your performance with our Testing and Analytics Package.